10
$\begingroup$

I've seen a lot of references to the fact that we can force a weakly compact to be indestructible by Cohen forcing, but after searching for a while haven't had any luck finding a proof of this. Any help with (a) what the proof looks like and (b) if this result is simply folklore, or if there is someone I should attribute this to, would be much appreciated!

$\endgroup$

2 Answers 2

12
$\begingroup$

Actually I think there is no known way to do this except for the Laver preparation to make a supercompact indestructible (assuming that by "indestructible" you mean indestructible via $\kappa$-directed closed forcings). This comes from Laver's aptly titled "Equiconsistencies at subcompact cardinals."

$\endgroup$
2
11
$\begingroup$

For indestructibility by adding Cohen subsets to $\kappa$, no extra strength is needed.

If $\kappa$ is weakly compact, then let $\newcommand\P{\mathbb{P}}\P$ be the length $\kappa$ Easton support iteration that uses $\text{Add}(\gamma,1)$ at every inaccessible $\gamma<\kappa$. Let $\newcommand\Q{\mathbb{Q}}\Q=\text{Add}(\kappa,1)$ be the forcing to add a Cohen subset to $\kappa$. In $V[G][g]$, generic for $\P*\Q$, I claim $\kappa$ remains weakly compact by the usual master condition lifting arguments. That is, for any $\kappa$-model $M$ and $j:M\to N$ in $V$, we can lift the embedding to $j:M[G][g]\to M[j(G)][j(g)]$.

The point now is that since adding two Cohen subsets is isomorphic to adding one, it follows that the weak compactness of $\kappa$ is indestructible by adding a Cohen subset over $V[G][g]$.

It follows that the weak compactness of $\kappa$ is indestructible in this model by $\text{Add}(\kappa,\theta)$ for any $\theta$. The reason is that any subset of $\kappa$ added by this forcing is added by a size $\kappa$ piece of it, using only $\kappa$ many of the coordiantes, and so one can thereby reduce to the case $\text{Add}(\kappa,1)$. That is, the embedding witness for any particular subset of $\kappa$ is already present in that extension.

If you want $\kappa$-c.c. indestructibility forcing, then don't force at every $\gamma<\kappa$, but do a lottery-preparation style forcing, and then you get that $\kappa$ is weakly compact in $V[G]$ and indestructible by Cohen forcing there.

$\endgroup$
3
  • 2
    $\begingroup$ Brilliant, thank you! Do you have a reference for this? Or should I take it to be a folklore result? $\endgroup$ Commented Apr 3 at 13:16
  • $\begingroup$ I had thought that this must have appeared in one of my papers, but I can't seem to find it now. I think this is a completely standard method. It is like the well-known case for measurability, except easier, since the models have size kappa and so the process of finding the generic filters for the lift proceeds in kappa many steps. The argument of my paper on unfoldability and GCH (cambridge.org/core/journals/journal-of-symbolic-logic/article/…) is a refinement of it to unfoldable cardinals. $\endgroup$ Commented Apr 3 at 21:21
  • $\begingroup$ I think the JSTOR link is probably better for old JSL papers, jstor.org/stable/2695100 $\endgroup$
    – Asaf Karagila
    Commented Apr 4 at 10:38

You must log in to answer this question.

Start asking to get answers

Find the answer to your question by asking.

Ask question

Explore related questions

See similar questions with these tags.